How Do You Apply Lagrange Multipliers to Optimize a Function with Constraints?

Click For Summary
To optimize the function f(x,y) = -2x^2 - 2xy + y^2 + 2 using Lagrange multipliers with the constraint 4x - y = 6, the Lagrangean must be set up incorporating the constraint. The first-order conditions involve calculating the partial derivatives ∂F/∂x and ∂F/∂y, which must equal zero. The discussion highlights a common mistake in deriving these derivatives, emphasizing the importance of correctly applying the method. The minimum values for x, y, λ, and f can be determined once the equations are correctly set up. Understanding these steps is crucial for successfully applying Lagrange multipliers in optimization problems.
peace89
Messages
8
Reaction score
0
Let f(x,y)= -2x^2-2xy+y^2+2 Use Lagrange multipliers to find the minimum of f subject to the constraint 4x-y = 6

∂F / ∂x =.....
i got -4x-2y+2y but i coming out as wrong what am i missing
∂F/ ∂Y= ...

The function f achieves its minimum, subject to the given constraint, where
x =
y =
λ=
f =
thank you
 
Last edited:
Physics news on Phys.org
Quoting from the forum rules:

NOTE: You MUST show that you have attempted to answer your question in order to receive help. You MUST make use of the homework template, which automatically appears when a new topic is created in the homework help forums. Once your question or problem has been responded to, do not go back and delete (or edit) your original post.
 
thanks just sign up here so don't know how things work here. learning
 
peace89 said:
Let f(x,y)= -2x^2-2xy+y^2+2 Use Lagrange multipliers to find the minimum of f subject to the constraint 4x-y = 6

∂F / ∂x =.....

∂F/ ∂Y= ...

The function f achieves its minimum, subject to the given constraint, where
x =
y =
λ=
f =
thank you

Set up your Lagrangean with the equality constraints.

This is a nonlinear program with equality constraints and thus it should be straightforward.

Apply your first order conditions.

Notice you don't need to check the second order conditions (Why?)
 
Question: A clock's minute hand has length 4 and its hour hand has length 3. What is the distance between the tips at the moment when it is increasing most rapidly?(Putnam Exam Question) Answer: Making assumption that both the hands moves at constant angular velocities, the answer is ## \sqrt{7} .## But don't you think this assumption is somewhat doubtful and wrong?

Similar threads

  • · Replies 8 ·
Replies
8
Views
2K
  • · Replies 16 ·
Replies
16
Views
3K
  • · Replies 2 ·
Replies
2
Views
2K
  • · Replies 6 ·
Replies
6
Views
2K
  • · Replies 18 ·
Replies
18
Views
3K
Replies
4
Views
2K
  • · Replies 9 ·
Replies
9
Views
2K
  • · Replies 10 ·
Replies
10
Views
2K
  • · Replies 5 ·
Replies
5
Views
2K
  • · Replies 8 ·
Replies
8
Views
3K